Đến nội dung

manhtuan00

manhtuan00

Đăng ký: 19-12-2016
Offline Đăng nhập: 21-03-2019 - 01:43
****-

#676723 Đề thi $Olympic$ $30/4$ lớp $11$ năm $2017...

Gửi bởi manhtuan00 trong 09-04-2017 - 12:56

Bài 6 : Xét tất cả các đường chéo có độ dài trên sẽ lập thành chu kì như sau : $A_1 \rightarrow A_4 \rightarrow A_7 \rightarrow A_{10} \rightarrow...... \rightarrow A_1$, chu kì có độ dài $38$.

Chu kì trên đi qua tất cả các đỉnh đúng 1 lần và chứa tất cả đường chéo cùng độ dài với $A_1A_4$ . Ta cần chứng minh số cạnh đen và số cạnh xanh trong chu kì trên là bằng nhau

Xét trong chu kì trên, ta gọi một đoạn là đoạn đen nếu tất cả các đỉnh trong đoạn đó màu đen, tương tự với đoạn xanh

Thật vậy, nếu đoạn đó chứa $n$ điểm thì sẽ có $n-1$ cạnh. Mà số đoạn đen bằng số đoạn xanh ( hiển nhiên ) , giả sử bằng $k$, khi đó số cạnh đen $=$ số cạnh xanh $= 19-k$. Ta có điều cần chứng minh




#676119 Tuần 1 tháng 4/2017: Chứng minh rằng $MN \parallel GL$.

Gửi bởi manhtuan00 trong 03-04-2017 - 19:48

Lời giải bài 1 : Gọi $U$ là giao điểm của 2 tiếp tuyến tại $B,C$ khi đó $K$ là hình chiếu của $O$ trên $AU$

Gọi $OK$ cắt $BC$ tại $S$ khi đó $AS$ là tiếp tuyến của $(O)$ tại $A$

Gọi $G'$ là giao điểm của $(DKH)$ với $BC$. Khi đó theo bài toán ở https://artofproblem...1407018p7885474 #6 thì $AG' \perp OH$ nên $G' \equiv G$ và $H,O,L$ thẳng hàng , $A,L,G$ thẳng hàng

$EF$ cắt $BC$ tại $T$ thì $T$ chính là trung điểm $GS$ theo bài toán trên

Do $EF \parallel AS$ do cùng vuông góc $AO$ nên $EF$ đi qua trung điểm $R$ của $AG$.

Ta chứng minh $MN,AD,EF$ đồng quy. Thật vậy, gọi $AH$ cắt $EF$ tại $V$. Thật vậy, theo Menelaus ta có $\frac{\overline{MF}}{\overline{MD}}.\frac{\overline{GD}}{\overline{GT}}.\frac{\overline{ET}}{\overline{EF}}= -1, \frac{\overline{ND}}{\overline{NE}}.\frac{\overline{GT}}{\overline{GD}}.\frac{\overline{FE}}{\overline{FT}} = -1$ nên $\frac{\overline{MF}}{\overline{MD}}.\frac{\overline{ND}}{\overline{NE}}.\frac{\overline{TE}}{\overline{TF}}= -1=\frac{\overline{MF}}{\overline{MD}}.\frac{\overline{ND}}{\overline{NE}}.\frac{\overline{VE}}{\overline{VF}}$ nên $M,V,N$ thẳng hàng

Gọi $MN$ cắt $BC$ tại $X$

Khi đó ta có : $V(MR,AG) = V(XT,DG) = N(XT,DG) = (VT,EF) = -1$ nên $MN \parallel AG$ hay $MN \parallel GL$. Ta có điều cần chứng minh

Hình gửi kèm

  • DSC_0123.JPG



#673814 Tuần 1 tháng 3/2017: Chứng minh $U,V,W$ thẳng hàng trên đường thẳng...

Gửi bởi manhtuan00 trong 09-03-2017 - 21:22

Lời giải của em ạ

Gọi $H_a,H_b,H_c$ là 3 chân đường cao thì $YZ$ là trung trưc $AH_a$

Khi đó ta có  $X,Y,Z$ chính là tâm ngoại tiếp các tam giác $\triangle AH_aD,\triangle BH_bE,\triangle CH_cF$
Xét phép biến hình $R_{\triangle} \circ I^A_{AB.AC}$, $D$ biến thành $H$, $K_a$ biến thành $M$ là trung điểm $BC$ và $H_a$ biến thành $A'$ đối xứng $A$ qua $O$. Ta có $M,H,A'$ thẳng hàng nên $A,H_a,K_a,D$ đồng viên
Tương tự có $B,H_b,K_b,E$ đồng viên và $C,H_c,K_c,F$ đồng viên
Có $\overline {HA}.\overline{HH_a}= \overline {HB}.\overline{HH_b}= \overline {HC}.\overline{HH_c}$ nên $H$ nằm trên trục đẳng phương của 3 đường tròn 
$(U),(V),(W)$
Lại có $\overline {LA}.\overline{LK_a}=\overline {LB}.\overline{LK_b}=\overline {LC}.\overline{LK_c}$ nên $L$ cũng nằm trên trục đẳng phương của 3 đường tròn
Vậy $HL$ nằm trên trục đẳng phương của 3 đường tròn này nên $HL \perp \overline{U,V,W}$

Hình gửi kèm

  • Untitled.png



#672910 $f(x-1-f(y))=3f(x)+2x+f(y), \forall x, y\in \mathbb{R}$

Gửi bởi manhtuan00 trong 27-02-2017 - 11:10

$f(x-1-f(y)) = 3f(x)+2x+f(y)$

$P(f(y),y) : f(-1) = 3f(f(y))+3f(y) \implies f(f(y)) = \frac{f(-1)}{3}-f(y)$

$P(2f(y)+1,y): f(f(y)) = 3f(2f(y)+1)+2(2f(y)+1)+f(y)$

$\implies f(2f(y)+1) = -2f(y) +\frac{f(-1)-6}{9}$

$P(3f(y)+2,y): f(2f(y)+1)=3f(3f(y)+2)+2(3f(y)+2)+f(y) \implies f(3f(y)+2) =-3f(y)+\frac{f(1)-42}{27}$

$P(3f(x)+2,y) : f(3f(x)+1-f(y)) = 3f(3f(x)+2)+2(3f(x)+2)+f(y) = f(y)-3f(x)+d$ trong đó $d$ là một hằng số

Thật vậy, Do $f(x-1-f(y)) - 3f(x) = 2x+f(y)$ nhận mọi giá trị trên $\mathbb R$ nên ta có thể thay $3f(x)+1-f(y)$ bởi $x$ nên 

$f(x) = a-x$ trong đó $a$ là một hằng số

Thay vào ta có $a = \frac{1}{2}$ nên $f(x) = -x +\frac{1}{2}$




#672847 Chứng minh đa thức bất khả quy

Gửi bởi manhtuan00 trong 26-02-2017 - 15:09

Cho $p$ là số nguyên tố có dạng $4k+3$. Chứng minh rằng đa thức $(x^2+1)^n+p$ bất khả quy trên $\mathbb Z[x]$




#672830 VMF's Marathon Hình học Olympic

Gửi bởi manhtuan00 trong 26-02-2017 - 11:51

Bài toán 177 :  Cho tứ giác nội tiếp $ABCD$, $AB$ cắt $CD$ tại $P$, $AD$ cắt $BC$ tại $Q$. Chứng minh rằng khoảng cách giữa trực tâm hai tam giác $APD$ và $AQB$ bằng khoảng cách giữa trực tâm hai tam giác $CQD$ và $BPC$




#672829 VMF's Marathon Hình học Olympic

Gửi bởi manhtuan00 trong 26-02-2017 - 11:44

Lời giải bài 176 : Gọi $M$ là chân đường phân giác ngoài góc $\angle A$, , $N$ là trung điểm $MD$, $NS$ là tiếp tuyến tới đường tròn $(T)$, $SD$ cắt trung trực $BC$ tại $X$, $R$ là trung điểm $BC$, $Z$ là trung điểm $SD$, $D'$ đối xứng $D$ qua $T$ suy ra $M,S,D'$ thẳng hàng.Gọi $L,L'$ là điểm chính giữa cung nhỏ , cung lớn $BC$. $K'$ đối xứng $L$ qua $K$ 

Ta có : $\frac{RX}{RD} = \frac{NZ}{ZD} = \frac{MD}{DD'}$ $= \frac{MD}{\frac{AD}{AL}.KL} = \frac{MD.AL}{AD.KL}= \frac{2R.AL}{AL'.K'L}$ $= \frac{2R}{K'L}.\frac{AL}{AL'} \implies RX =RD. \frac{2R}{K'L}.\frac{AL}{AL'} = RL .\frac{2R}{K'L}$ không đổi nên $X$ cố định

Thật vậy , ta có $DS.DX = DR.DM = DB.DC$ nên tứ giác $SBXC$ nội tiếp

Lại có $NS^2 = ND^2 = NM^2 = NB.NC$ nên $NS$ cũng là tiếp tuyến tới đường tròn $(SBXC)$. Mà điểm $X$ cố định nên đường tròn $(SBXC)$ cố định nên $(T)$ tiếp xúc với đường tròn cố định là $(BXC)$

 

Hình gửi kèm

  • 600693_126327790906289_1017420421_n.jpg



#672711 VMF's Marathon Hình học Olympic

Gửi bởi manhtuan00 trong 25-02-2017 - 10:55

Bài toán 173. Cho tứ giác $ABCD$ nội tiếp đường tròn $(O)$, $E$ là giao điểm của $AB$ với $CD$, $F$ là giao điểm của $AD$ với $BC$ . $(AEF)$ cắt $(O)$ tại $P$, $(CEF)$ cắt $O$ tại $Q$. $AC$ cắt $BD$ tại $I$. Chứng minh rằng $P,I,Q$ thẳng hàng.




#672452 VMF's Marathon Hình học Olympic

Gửi bởi manhtuan00 trong 23-02-2017 - 00:17

Lời giải bài 172 : 

Trước tiên ta chứng min bổ đề sau : 

Bổ đề : Cho $\triangle ABC$. Đường tròn $(X)$ bất kì qua $A$ tiếp xúc với đường tròn Euler cắt $AB,AC$ tại $M,N$. Chứng minh rằng đường tròn $(XMN)$ tiếp xúc với đường trung bình của $\triangle ABC$

 

Chứng minh : Xét phép nghịch đảo $I^A_k$, bài toán trên trở thành : Cho $\triangle ABC$ nội tiếp $(O)$. Đường thẳng bất kì tiếp xúc $(BOC)$ cắt $CA,AB$ tại $M,N$. $D$ đối xứng $A$ qua $MN$. Khi đí $(DMN)$ tiếp xúc $(O)$ ( đây chính là bài toán SMO 2016 ở đây https://www.artofpro...1220645p6102531 )

 

Áp dụng : Gọi $T$ là trung điểm $MH$

Theo định lý Menelaus ta có : $1 = \frac{\overline{DQ}}{\overline{DP}}.\frac{\overline{NP}}{\overline{NA}}.\frac{\overline{HA}}{\overline{HQ}}= \frac{\overline{DQ}}{\overline{DP}}.\frac{\overline{MP}}{\overline{MC}}.2=\frac{\overline{DQ}}{\overline{DP}}\frac{\overline{MP}}{\overline{MH}} \implies \frac{\overline{DQ}}{\overline{DP}}=\frac{\overline{MH}}{\overline{MP}}$ nên $MD \perp BC$

Ta có đường trung bình của $\triangle DAH$ tiếp xúc $(XMN)$ theo bổ đề, mà đường trung bình này chính là đường thẳng qua $T$ vuông góc $BC$ nên $(XMN)$ tiếp xúc đường thẳng cố định là đường qua trung điểm $MH$ vuông góc $BC$

Hình gửi kèm

  • Bài 4.png



#670608 Tuần 1 tháng 2/2017: $QR$ đi qua điểm cố định khi $P$ di...

Gửi bởi manhtuan00 trong 07-02-2017 - 01:30

Lời giải của em ạ 

Gọi $N$ đối xứng $P$ qua $O$ thì $N,E,B$ thẳng hàng và $N,F,C$ thẳng hàng. $BR,CR$ cắt $NC,NB$ tại $U,V$

$\blacksquare$ Ta chứng minh $AN,UV,BC$ đồng quy

$\angle PCR = \angle PBR$ nên $\angle VCN = \angle UBN \implies U,V,B,C$ đồng viên

$UV$ cắt $BC$ tại $T$ . Gọi $I$ là tâm $(UVBC)$. Theo định lý Brocard, $TR \perp NI$ tại $X$, $AR \perp TI$ tại $Y$

Xét nghịch đảo $ I^N_{NV.NB} : B \rightarrow V, C \rightarrow U,R \rightarrow Y, I \rightarrow X, P \rightarrow W ( W \equiv AO \cap UV)$

Thật vậy ta có $N,W,X,Y$ đồng viên trên đường tròn đường kính $NT$ nên $R,I,P$ thẳng hàng. Từ đây có $NA \perp RI$. Theo Brocard thì $NT \perp RI$ nên $N,T,A$ thẳng hàng.

$\blacksquare$ Ta chứng minh $QR$ đi qua điểm cố định 

Có $N(RT,BC) = F(ET,BC) = -1$ nên $N,Q,R$ thẳng hàng. $RQ$ cắt $(O)$ tại $L$ thì $(AL,BC) = N(AL,BC) = (TQ,BC) = -1 \implies L$ cố định. Vậy $QR$ đi qua $L$ cố định với $L$ thỏa mãn tứ giác $ABLC$ điều hòa




#670524 $f(x^{2} + f(y)) = \frac{f^{2}(x)}{2} + 4y$

Gửi bởi manhtuan00 trong 31-01-2017 - 00:21

$f(x^2+f(y)) = \frac{f^2(x)}{2}+4y$

$P(0,y):f(f(y)) = \frac{f^2(0)}{2}+4y \implies f$ song ánh

$P(-x,y) \implies f^2(x) = f^2(-x) \implies f(-x) = -f(x) \implies f(0) = 0$

$P(0,y) : f(x^2) = \frac{f^2(x)}{2}$

$P(x,0) : f(f(y)) = 4y$

$\implies f(x^2+f(y))= f(x^2)+f(f(y))$

Do $f$ song ánh nên có thể thay $f(y)$ bởi $y$ nên

$f(x^2+y) = f(x^2)+f(y)$

Do $f(x^2) = \frac{f^2(x)}{2} >0$ nên $f(x^2+y) > f(y)$. Vậy $f$ tăng ngặt

Lại có $f$ cộng tính nên $f(x) = ax \implies f(x) = 2x$

 




#670522 Tuần 5 tháng 1/2017: $AR$ và trung trực $MN$ cắt nhau trê...

Gửi bởi manhtuan00 trong 30-01-2017 - 23:51

Lời giải của em ạ

Gọi $S$ là chân đường phân giác ngoài góc $\angle A$ của $\triangle ABC$. khi đó $S,P,Q$ thẳng hàng

$AR$ cắt $(I)$ tại $T$ . Ta cần chứng minh $T$ nằm trên trung trực $MN$

$AI$ cắt $(I)$ tại $G$. Ta có $\angle NAG = \angle FJG$ nên $A,N,G,D$ đồng viên, tương tự ta có $A,M,D,G$ đồng viên nên $A,M,N,D,G$ đồng viên trên $(O')$

Ta có $\angle NAG = \angle FJG = \angle EJG = \angle MAG$ nên $G$ là điểm chính giữa cung của đường tròn $(O')$

Do $JG$ là đường kính của $(I)$ nên $TJ \perp TG$ . Ta sẽ chứng minh $JT \parallel MN$ từ đó suy ra $TG \perp MN$ thì $TG$ là trung trực $MN$

Gọi $U$ là trung điểm $EF$, $V$ là giao điểm của $AS$ và $ID$

Có $A(IV,RD) = (IV,RD) = -1$ nên $AI$ là phân giác $\angle DAR$.

Gọi $H$ là giao của $DU$ với $(I)$. Khi đó ta có $\triangle FTA = \triangle EHA \sim \triangle DEA$ nên $AT,AD$ đẳng giác trong $\angle EAF$. Khi đó ta có $H \equiv T$. Vậy $T,U,D$ thẳng hàng

Ta có $IU.IA = ID^2$ nên $\angle IAT = \angle IAD = \angle UDI$ nên $A,T,I,D$ đồng viên, tức là $A,T,I,D,S$ đồng viên $\implies \angle STI = 90^{\circ} \implies ST$ là tiếp tuyến của $(I)$

Ta có $(NM,DG) = (NM,NG)+(NG,DG) = (DM,DG)+(AN,AD) = (JE,JG)+(AN,AG)+(AG,AD) = (JE,JG)+(JF,JG)+(AG,AD)= (AG,AD)$

Lại có $(JT,DG) = (JT,TS) + (TS,DS)+ (DS,DT)+(DT,DG) = (GJ,GT)+(AT,AD)+(DS,DT)+(TJ,GJ) = (AT,AD)+(DS,DT)-(TJ,TG)= (DS,DT)-(TJ,TG)+2(AG,AD) = (DS,DT)-(TJ,TG)+(DT,DI)+(AG,AD)=(DS,DI) - (TJ,TG)+(AG,AD) = (AG,AD)$

$\implies (NM,DG) = (JT,DG) \implies JT \parallel MN$. Ta có điều cần chứng minh

Hình gửi kèm

  • Untitled.png



#669464 Tuần 4 tháng 1 năm 2017 : $JL\perp ON$

Gửi bởi manhtuan00 trong 22-01-2017 - 22:05

Gọi $W,R$ lần lượt là chân đường phân giác trong, chân đường phân giác ngoài góc $\angle A$. $T$ là chân tiếp tuyến tại $A$ nên $T$ là trung điểm $RW$.

Ta sẽ chứng minh $A,I,D,L$ đồng viên

Thật vậy, Gọi $S,Z$ là trung điểm cung lớn, cung nhỏ $BC$. 

Gọi $U$ là giao điểm của $(AEIF)$ với $(O)$. $X$ là giao của $EF$ với $BC$

Theo 1 tính chất quen thuộc thì $UD$ là phân giác $\angle BUC$ nên $U,D,Z$ thẳng hàng

$(XD,BC) = -1$ nên $UX$ là phân giác ngoài $\angle BUC$ nên $UX$ đi qua $S \implies D$ là trực tâm $\triangle SXZ$ nên $XZ \perp DS$

Từ đây ta có $D,L,S$ thẳng hàng nên $\angle LAI = \angle LSZ = \angle IDS$ nên $A,I,D,L$ đồng viên

Gọi $P'$ là tâm $ADIL$. Do $A,D,I,R$ đồng viên nên $A,D,I,R,L$ đồng viên nên $P'$ là trung điểm $IR$ 

$\implies TP' \parallel AI$ nên $p' \equiv P$ tức là $P$ là tâm $(ARLDI)$

Gọi $Z'$ đối xứng $Z$ qua $N$ thì $Z'PZM$ là hình bình hành nên $PZ' = ZM = \frac{IL}{2} \implies Z'$ là trung điểm $RL$

Gọi $V$ là trung điểm $RS$. Ta có $\angle VRZ' = \angle RIZ$ và $\angle RVZ' = \angle RSL = \angle AZL$ nên $\triangle RZ'V \sim \triangle ILZ$

Từ đây ta có $\frac{RZ'}{RV}= \frac{IL}{IZ} \implies  \frac{RZ'}{RS}= \frac{IL}{IJ}$ nên $\triangle ILJ \sim \triangle RSZ'$. Từ đây có $\angle RSZ' = \angle AJL$ mà $AJ \perp SR$ nên $LJ \perp SZ'$

$ON \parallel SZ'$ do là đường trung bình nên $ON \perp LJ$  $\square$

Hình gửi kèm

  • Untitled.png



#669403 Bài toán T12/471 THTT

Gửi bởi manhtuan00 trong 22-01-2017 - 19:11

Lời giải của em ạ 
Gọi $X,Y$ là điểm chính giữa cung $AC,AB$ thì $Q,P$ đối xứng $A$ qua $X,Y$

Từ đây ta có $QM$ đi qua tâm bàng tiếp $I_b$ và $PN$ đi qua tâm bàng tiếp $I_c$

$I_bI_C$ cắt $BC$ tại $T$, $S$ là chân đường phân giác trong góc $\angle A$ thì $(TRMN)= (TAI_bI_c) = -1$ nên $AR$ cũng là phân giác $\angle MAN$




#669147 VMF's Marathon Hình học Olympic

Gửi bởi manhtuan00 trong 20-01-2017 - 23:33

Bài toán 140 : (sách) Cho tứ giác $ABCD$ nội tiếp. Giả sử rằng tồn tại điểm $X$ để $\angle XAD = \angle XBA = \angle XCB = \angle XDC$. Chứng minh rằng tứ giác $ABCD$ điều hòa